Limit of Multivariable Function: x^2+y^2+2xy

Click For Summary
The limit of the function (x^2+y^2+2xy)/(x^2+y^2) as (x,y) approaches (0,0) is analyzed using polar coordinates, simplifying to 1 + 2cos(theta)sin(theta). Calculations show that the limit varies based on the approach direction, yielding different results: 1 when approaching along the x-axis and 2 along the line x=t, y=t. This variation indicates that the limit depends on the angle theta, suggesting that the limit does not exist. Therefore, if different approaches yield different limits, it confirms the non-existence of the limit at that point.
fogel1497
Messages
10
Reaction score
0

Homework Statement



Find the limit of:
(x^2+y^2+2xy)/(x^2+y^2)

Homework Equations


x = r*cos(theta)
y= r*sin(theta)

The Attempt at a Solution



So what I did was change to polar coordinates. Then it simplifies to:

(r^2 + 2r^2cos(theta)sin(theta) )/r^2

Factoring out an r^2 from everything you get:
1 + 2cos(theta)sin(theta)

And the limit as r and theta go to zero would appear to be 1. However if I plug in numbers very close to zero on my calculator it tells me that the limit is 2. Little help?
 
Physics news on Phys.org
Your limit depends on theta. That means the limit is different as you approach (0,0) in different ways. E.g. if you approach along the x axis, x=t, y=0 (theta=0), you get 1. If you approach along a line x=t, y=t, (theta=pi/4) you get 2. What does this tell you about the existence of the limit?
 
It doesn't exist? So when i do that method if I find i have thetas in my answer then the limit does not exist?
 
If you get different answers for different values of theta, yes, the limit does not exist.
 
Question: A clock's minute hand has length 4 and its hour hand has length 3. What is the distance between the tips at the moment when it is increasing most rapidly?(Putnam Exam Question) Answer: Making assumption that both the hands moves at constant angular velocities, the answer is ## \sqrt{7} .## But don't you think this assumption is somewhat doubtful and wrong?

Similar threads

  • · Replies 20 ·
Replies
20
Views
2K
  • · Replies 8 ·
Replies
8
Views
2K
Replies
3
Views
2K
  • · Replies 19 ·
Replies
19
Views
2K
  • · Replies 3 ·
Replies
3
Views
2K
  • · Replies 14 ·
Replies
14
Views
2K
  • · Replies 3 ·
Replies
3
Views
2K
  • · Replies 1 ·
Replies
1
Views
2K
  • · Replies 5 ·
Replies
5
Views
2K
Replies
5
Views
2K